LSAT and Law School Admissions Forum

Get expert LSAT preparation and law school admissions advice from PowerScore Test Preparation.

 karen_k
  • Posts: 35
  • Joined: Sep 24, 2015
|
#19962
Hi,

I was stuck between answer choice A and B on this question and ended up choosing B. I understand how A is correct in that the property of being patterns of electronic signals is shown to have a positive result but also a possible negative result and that would therefore lead to answer choice A. However, I am still confused as to why exactly B is wrong because the two answer choices seem very similar to me. Thank you!
 jeff.wren
PowerScore Staff
  • PowerScore Staff
  • Posts: 26
  • Joined: Jul 04, 2015
|
#19965
Hi Karen,

Thanks for your question. Answer choice (B) is tricky. The key difference between (A) and (B) is that (B) mentions creating more problems than it solves. The stimulus never compares or quantifies the advantages and problems. This answer goes too far.

I hope this helps.

Best,
Jeff
 karen_k
  • Posts: 35
  • Joined: Sep 24, 2015
|
#19977
Thank you for the explanation!

Get the most out of your LSAT Prep Plus subscription.

Analyze and track your performance with our Testing and Analytics Package.